1. The Causal system given below is __________ H(z) = 6 + z-1 - z-2





Ask Your Doubts Here

Type in
(Press Ctrl+g to toggle between English and the chosen language)

Comments

  • By: guest on 02 Jun 2017 12.54 am
    Minimum phase H(z) = 6 + z-1 - z-1 6z2 + z - 1 = 0 ∴ We have zeros at z = 1/3 and z = - 1/2 ∴ All the zeroes lies inside the unit circle, it is a min. phase system.
Show Similar Question And Answers
QA->A statement followed by two assumptions I and II is given. You have to consider the statement to be true even if it seems to be at variance from commonly known facts. You are to decide which of the given assumptions can definitely be drawn from the given statement. Indicate which one of the four given alternatives is correct ? Statement : If more encouragement is given to Sports, Indians will win more gold medals at the Olympic Games. Assumptions : I. Indians do not win gold medals. II. More enc....
QA->Two statements are given followed by two conclusions I and II. You have to consider the two statements to be true even if they seem to be at variance from commonly known facts. You have to decide which one of the given conclusions is definitely drawn from the given statements. Statement : All virtuous persons are happy. No unhappy person is virtuous. Conclusions : I. Happiness is related to virtue II. Unhappy person is not virtuous.....
QA->Who discovered the causal organism of the disease Anthrax?....
QA->The causal organism of polio is?....
QA->Select Ore of Aluminium given below?....
MCQ->The transfer function of a discrete time LTI system is given by Consider the following statements: S1: The system is stable and causal for ROC: |z| > 1/2 S2: The system is stable but not causal for ROC: |z| < 1/4 S3: The system is neither stable nor causal for ROC: 1/4 < |z| < 1/2 Which one of the following statements is valid?....
MCQ->Consider a system whose input x and output y are related by the equation y(t) = x(t - t )h(2t )dt where h(t) is shown in the graph which of the following four properties are possessed by the system? BIBO: Bounded input gives a bounded output Causal: The system is causal LP: The system is low pass LTI: The system is linear and time invariant ....
MCQ->The Causal system given below is __________ H(z) = 6 + z-1 - z-2....
MCQ->The causal system given below is __________ ....
MCQ->Pick out thể one word for - a secret arrangement....
Terms And Service:We do not guarantee the accuracy of available data ..We Provide Information On Public Data.. Please consult an expert before using this data for commercial or personal use | Powered By:Omega Web Solutions
© 2002-2017 Omega Education PVT LTD...Privacy | Terms And Conditions
Question ANSWER With Solution